LSAT and Law School Admissions Forum

Get expert LSAT preparation and law school admissions advice from PowerScore Test Preparation.

User avatar
 Dave Killoran
PowerScore Staff
  • PowerScore Staff
  • Posts: 5852
  • Joined: Mar 25, 2011
|
#80548
This game is also discussed in our Podcast: LSAT Podcast Episode 70: The May 2020 LSAT-Flex Logic Games Section

Complete Question Explanation
(The complete setup for this game can be found here: viewtopic.php?t=33052)

The correct answer choice is (C).

The question stem places H on day 2, which immediately triggers the second rule, placing H on day 7 as well:


  • ___ ..... _H_ ..... ___ ..... ___ ..... ___ ..... ___ ..... _H_ ..... ___
     1 .....     2 .....    3 .....    4 .....     5 .....    6 .....      7 .....   8

The placement of H then triggers the fifth rule, forcing J ahead of the first H. In this case, that forces J into day 1. And, via the first rule, J must also then perform on day 5:

  • _J_ ..... _H_ ..... ___ ..... ___ ..... _J_ ..... ___ ..... _H_ ..... ___
     1 .....     2 .....    3 .....    4 .....     5 .....    6 .....      7 .....   8

These rule connections eliminate answer choices (A) and (E) since J must be performed on days 1 and 5. Answer choice (B) is also eliminated, since J has already been performed twice, and thus could not be performed a third time, on day 3.

The remaining four plays—G, L, M, and O—must now appear a single time each on days 3, 4, 6, and 8, not necessarily in that order. We still have two other rules to consider (the third and fourth), however, and both involve M so immediately focus on M!

Per the fourth rule, there must be an HM block, and thus M must be performed on either day 3 or day 8 in this question. Answer choice (D), which attempts to performed M on day 7, thus cannot occur and can be eliminated.

The only answer choice remaining is (C), which addresses L. L is a random and not surprisingly can be performed on day 8. Thus, answer choice (C) is correct.


Answer choice (A): Since J must be performed on day 1, this answer choice cannot occur.

Answer choice (B): From the numerical distribution, J can only perform a maximum of twice. since J performs on days 1 and 5 in this questions, J cannot then perform a third time, on day 3. Thus, this answer choice is incorrect.

Answer choice (C): This is the correct answer choice.

Answer choice (D): Per the fourth rule, there must be an HM block, and thus M must be performed on either day 3 or day 8 in this question. This answer, which attempts to have M performed on day 7, thus cannot occur and can be eliminated.

Answer choice (E): Since J must be performed on day 5, this answer choice cannot occur.
User avatar
 christealeaf
  • Posts: 1
  • Joined: Jan 05, 2021
|
#82997
Hi, i'm wondering how to diagram this question and how to approach these types of questions (more spaces than elements)?
 Rachael Wilkenfeld
PowerScore Staff
  • PowerScore Staff
  • Posts: 1358
  • Joined: Dec 15, 2011
|
#83041
Great question christealeaf.

This is an unbalanced game, specifically underfunded. That means we have more spaces than we have variables. Our rules don't give us any direct information about how many times a particular play can be performed. We would start then by making a list of possible numerical distributions for this game. This will help us know how many times a play could potentially be performed.

So at minimum, each play needs to be performed once. That distribution would look like

1-1-1-1-1-1

where each 1 represents the number of times a play is performed.

We still then have two performances to fill. There are two ways to potentially do this. Either putting both extras in one play, or splitting the two extras across two plays.

3-1-1-1-1-1

or

2-2-1-1-1-1

Those are the only two ways to distribute the plays over the required 8 days.

In this case, we get more information. Rule 1 says that the play performed on Day 1 is also on Day 5, and Rule 2 tells us that the play performed on Day 2 is also on Day 7.

Now we have our distribution. If two different plays are each on two different days, that means we are in the realm of 2-2-1-1-1-1, or two plays on two days, the rest on only one day.

Now let's look at the actual question. It says that H is on day 2. That means it also has to be on day 7. And, because J has to be before our first H, J has to be on days 1 and 5. Our two doubles are H and J, and all the other plays are only performed once.

What does our set up look like?

J H _ _ J _ H _

Answer choice (A): G cannot be on 1 because J has to be before H

Answer choice (B): J cannot be on 3 because J has to be on 1 and 5.

Answer choice (C): This is our correct answer. It does not violate any rules.

Answer choice (D): M cannot be on day 4 because M has to be directly after an H. That means that M has to be either 3 or 8.

Answer choice (E): O cannot be on day 5 because J is on day 5.

Hope that helps!

Get the most out of your LSAT Prep Plus subscription.

Analyze and track your performance with our Testing and Analytics Package.